Sylvester rank inequality: $operatornamerank A + operatornamerankB leq operatornamerank AB + n$Rank of matrices, prove inequalityHow to prove and interpret $operatornamerank(AB) leq operatornamemin(operatornamerank(A), operatornamerank(B))$?rank(AB) = rank(A) if B is invertibleprove that $textrank(AB)getextrank(A)+textrank(B)-n.$Matrices and rank inequalityHow to show that $Rank(AB)geq Rank(A)+Rank(B)-n$Proof of Sylvester rank inequalityFor nonzeros $A,B,Cin M_n(mathbbR)$, $ABC=0$. Show $operatornamerank(A)+operatornamerank(B)+operatornamerank(C)le 2n$Let A be $ m times n$ and $B$ be $n times p$, and suppose $AB = 0$. Explain why $mathrmrank(A) + mathrmrank(B) leq n.$Linear transformation over two vector spacesShow that $operatornamerank(A+B) leq operatornamerank(A) + operatornamerank(B)$On the rank inequality $operatornamerank(A)+operatornamerank(A^3)geq2operatornamerank(A^2)$Rank of the product of two full rank matricesSimultaneous Rank MinimizationProof that $operatornamerank(SAT)= operatornamerank(A)$Prove that $operatornamerank (f) + operatornamerank (g) -dim Wleq operatornamerank(gcirc f)$.How to prove $operatornamerank(XGY^T) = operatornamerank(G)$?Prove that $det(AB-BA)=0$Can the limit of a sequence of matrices of fixed rank have higher rank?Does $operatornamerank (A^2) = operatornamerank (A)$ for any matrix $Ain operatornameMat_n times n$?

Arthur Somervell: 1000 Exercises - Meaning of this notation

What defenses are there against being summoned by the Gate spell?

Theorems that impeded progress

I’m planning on buying a laser printer but concerned about the life cycle of toner in the machine

Dragon forelimb placement

"to be prejudice towards/against someone" vs "to be prejudiced against/towards someone"

What's the output of a record cartridge playing an out-of-speed record

Why doesn't Newton's third law mean a person bounces back to where they started when they hit the ground?

Email Account under attack (really) - anything I can do?

TGV timetables / schedules?

How old can references or sources in a thesis be?

Why "Having chlorophyll without photosynthesis is actually very dangerous" and "like living with a bomb"?

To string or not to string

Is a tag line useful on a cover?

How can bays and straits be determined in a procedurally generated map?

Modeling an IPv4 Address

Risk of getting Chronic Wasting Disease (CWD) in the United States?

Problem of parity - Can we draw a closed path made up of 20 line segments...

A newer friend of my brother's gave him a load of baseball cards that are supposedly extremely valuable. Is this a scam?

Have astronauts in space suits ever taken selfies? If so, how?

Mathematical cryptic clues

What would happen to a modern skyscraper if it rains micro blackholes?

How much RAM could one put in a typical 80386 setup?

Why can't I see bouncing of a switch on an oscilloscope?



Sylvester rank inequality: $operatornamerank A + operatornamerankB leq operatornamerank AB + n$


Rank of matrices, prove inequalityHow to prove and interpret $operatornamerank(AB) leq operatornamemin(operatornamerank(A), operatornamerank(B))$?rank(AB) = rank(A) if B is invertibleprove that $textrank(AB)getextrank(A)+textrank(B)-n.$Matrices and rank inequalityHow to show that $Rank(AB)geq Rank(A)+Rank(B)-n$Proof of Sylvester rank inequalityFor nonzeros $A,B,Cin M_n(mathbbR)$, $ABC=0$. Show $operatornamerank(A)+operatornamerank(B)+operatornamerank(C)le 2n$Let A be $ m times n$ and $B$ be $n times p$, and suppose $AB = 0$. Explain why $mathrmrank(A) + mathrmrank(B) leq n.$Linear transformation over two vector spacesShow that $operatornamerank(A+B) leq operatornamerank(A) + operatornamerank(B)$On the rank inequality $operatornamerank(A)+operatornamerank(A^3)geq2operatornamerank(A^2)$Rank of the product of two full rank matricesSimultaneous Rank MinimizationProof that $operatornamerank(SAT)= operatornamerank(A)$Prove that $operatornamerank (f) + operatornamerank (g) -dim Wleq operatornamerank(gcirc f)$.How to prove $operatornamerank(XGY^T) = operatornamerank(G)$?Prove that $det(AB-BA)=0$Can the limit of a sequence of matrices of fixed rank have higher rank?Does $operatornamerank (A^2) = operatornamerank (A)$ for any matrix $Ain operatornameMat_n times n$?













32












$begingroup$



If $A$ and $B$ are two matrices of the same order $n$, then
$$ operatornamerank A + operatornamerankB leq operatornamerank AB + n. $$




I don't know how to start proving this inequality. I would be very pleased if someone helps me. Thanks!



Edit I. Rank of $A$ is the same of the equivalent matrix $A' =beginpmatrixI_r & 0 \ 0 & 0endpmatrix$. Analogously for $B$, ranks of $A$ and $B$ are $r,sleq n$. Hence, since $operatornamerankAB = minr,s$, then $r+sleq minr,s + n$. (This is not correct since $operatornamerank AB leq minr,s$.



Edit II. A discussion on the rank of a product of $H_f(A)$ and $H_c(B)$ would correct this, but I don't know how to formalize that $operatornamerankH_f(A) +operatornamerankH_c(B) - n leq operatornamerank[H_f(A)H_c(B)]$.










share|cite|improve this question











$endgroup$











  • $begingroup$
    This might be of help: math.stackexchange.com/questions/978/…
    $endgroup$
    – Git Gud
    Feb 9 '13 at 19:54











  • $begingroup$
    @GitGud Not really, I knew that but doesn't solves anything. Thanks anyway.
    $endgroup$
    – Ben Ward
    Feb 9 '13 at 20:04











  • $begingroup$
    See also: math.stackexchange.com/questions/269474/…
    $endgroup$
    – Martin Sleziak
    May 28 '16 at 14:02















32












$begingroup$



If $A$ and $B$ are two matrices of the same order $n$, then
$$ operatornamerank A + operatornamerankB leq operatornamerank AB + n. $$




I don't know how to start proving this inequality. I would be very pleased if someone helps me. Thanks!



Edit I. Rank of $A$ is the same of the equivalent matrix $A' =beginpmatrixI_r & 0 \ 0 & 0endpmatrix$. Analogously for $B$, ranks of $A$ and $B$ are $r,sleq n$. Hence, since $operatornamerankAB = minr,s$, then $r+sleq minr,s + n$. (This is not correct since $operatornamerank AB leq minr,s$.



Edit II. A discussion on the rank of a product of $H_f(A)$ and $H_c(B)$ would correct this, but I don't know how to formalize that $operatornamerankH_f(A) +operatornamerankH_c(B) - n leq operatornamerank[H_f(A)H_c(B)]$.










share|cite|improve this question











$endgroup$











  • $begingroup$
    This might be of help: math.stackexchange.com/questions/978/…
    $endgroup$
    – Git Gud
    Feb 9 '13 at 19:54











  • $begingroup$
    @GitGud Not really, I knew that but doesn't solves anything. Thanks anyway.
    $endgroup$
    – Ben Ward
    Feb 9 '13 at 20:04











  • $begingroup$
    See also: math.stackexchange.com/questions/269474/…
    $endgroup$
    – Martin Sleziak
    May 28 '16 at 14:02













32












32








32


23



$begingroup$



If $A$ and $B$ are two matrices of the same order $n$, then
$$ operatornamerank A + operatornamerankB leq operatornamerank AB + n. $$




I don't know how to start proving this inequality. I would be very pleased if someone helps me. Thanks!



Edit I. Rank of $A$ is the same of the equivalent matrix $A' =beginpmatrixI_r & 0 \ 0 & 0endpmatrix$. Analogously for $B$, ranks of $A$ and $B$ are $r,sleq n$. Hence, since $operatornamerankAB = minr,s$, then $r+sleq minr,s + n$. (This is not correct since $operatornamerank AB leq minr,s$.



Edit II. A discussion on the rank of a product of $H_f(A)$ and $H_c(B)$ would correct this, but I don't know how to formalize that $operatornamerankH_f(A) +operatornamerankH_c(B) - n leq operatornamerank[H_f(A)H_c(B)]$.










share|cite|improve this question











$endgroup$





If $A$ and $B$ are two matrices of the same order $n$, then
$$ operatornamerank A + operatornamerankB leq operatornamerank AB + n. $$




I don't know how to start proving this inequality. I would be very pleased if someone helps me. Thanks!



Edit I. Rank of $A$ is the same of the equivalent matrix $A' =beginpmatrixI_r & 0 \ 0 & 0endpmatrix$. Analogously for $B$, ranks of $A$ and $B$ are $r,sleq n$. Hence, since $operatornamerankAB = minr,s$, then $r+sleq minr,s + n$. (This is not correct since $operatornamerank AB leq minr,s$.



Edit II. A discussion on the rank of a product of $H_f(A)$ and $H_c(B)$ would correct this, but I don't know how to formalize that $operatornamerankH_f(A) +operatornamerankH_c(B) - n leq operatornamerank[H_f(A)H_c(B)]$.







linear-algebra matrices inequality matrix-rank






share|cite|improve this question















share|cite|improve this question













share|cite|improve this question




share|cite|improve this question








edited Apr 22 '17 at 9:39









Martin Sleziak

45k10122277




45k10122277










asked Feb 9 '13 at 17:07









Ben WardBen Ward

2471312




2471312











  • $begingroup$
    This might be of help: math.stackexchange.com/questions/978/…
    $endgroup$
    – Git Gud
    Feb 9 '13 at 19:54











  • $begingroup$
    @GitGud Not really, I knew that but doesn't solves anything. Thanks anyway.
    $endgroup$
    – Ben Ward
    Feb 9 '13 at 20:04











  • $begingroup$
    See also: math.stackexchange.com/questions/269474/…
    $endgroup$
    – Martin Sleziak
    May 28 '16 at 14:02
















  • $begingroup$
    This might be of help: math.stackexchange.com/questions/978/…
    $endgroup$
    – Git Gud
    Feb 9 '13 at 19:54











  • $begingroup$
    @GitGud Not really, I knew that but doesn't solves anything. Thanks anyway.
    $endgroup$
    – Ben Ward
    Feb 9 '13 at 20:04











  • $begingroup$
    See also: math.stackexchange.com/questions/269474/…
    $endgroup$
    – Martin Sleziak
    May 28 '16 at 14:02















$begingroup$
This might be of help: math.stackexchange.com/questions/978/…
$endgroup$
– Git Gud
Feb 9 '13 at 19:54





$begingroup$
This might be of help: math.stackexchange.com/questions/978/…
$endgroup$
– Git Gud
Feb 9 '13 at 19:54













$begingroup$
@GitGud Not really, I knew that but doesn't solves anything. Thanks anyway.
$endgroup$
– Ben Ward
Feb 9 '13 at 20:04





$begingroup$
@GitGud Not really, I knew that but doesn't solves anything. Thanks anyway.
$endgroup$
– Ben Ward
Feb 9 '13 at 20:04













$begingroup$
See also: math.stackexchange.com/questions/269474/…
$endgroup$
– Martin Sleziak
May 28 '16 at 14:02




$begingroup$
See also: math.stackexchange.com/questions/269474/…
$endgroup$
– Martin Sleziak
May 28 '16 at 14:02










6 Answers
6






active

oldest

votes


















30












$begingroup$

Subtract $2n$ from both sides and then multiply by $-1$; this gives the equivalent inequality $defrkoperatornamerank(n-rk A)+(n-rk B)geq n-rk(AB)$. In other words (by the rank-nullity theorem) we must show that the sums of the dimensions of the kernels of $A$ and of $B$ gives at least the dimension of the kernel of $AB$. Intuititively vectors that get killed by $AB$ either get killed by $B$ or (later) by $A$, and the dimension of $ker(AB)$ therefore cannot exceed $dimker A+dimker B$. Here is how to make that precise.



One has $ker Bsubseteq ker(AB)$, so if one restricts (the linear map with matrix) $B$ to $ker(AB)$, giving a map $tilde B:ker(AB)to K^n$, one sees that $ker(tilde B)=ker(B)$ (both inclusions are obvious). Since the image of $tilde B$ is contained in $ker A$ by the definition of $ker(AB)$, one has $rktilde Bleqdimker A$. Now rank-nullity applied to $tilde B$ gives $$dimker(AB)=dimkertilde B+rktilde Bleqdimker B+dimker A,$$ as desired.






share|cite|improve this answer











$endgroup$












  • $begingroup$
    i cant understand how or why you fixed $(n - rank(A)) + (n - rank(B))leq (n - rank(AB)) $ as according to the rank-nullity theorem, this is exactly: $N(A) + N(B) = N(AB)$, i mean, that wouldn't be correct to use $leq$
    $endgroup$
    – Jneven
    Jul 17 '18 at 9:20











  • $begingroup$
    @jneven: As the text says, the new inequality is equivalent to the old one. It still needs to be proved, which I then do.
    $endgroup$
    – Marc van Leeuwen
    Jul 17 '18 at 11:49











  • $begingroup$
    Could you please explain a bit more detail on "restricts matrix to a map"? I searched on the web but only found "restricted to a subspace / domain". Does these have same meaning? Thanks.
    $endgroup$
    – Page David
    Mar 29 at 14:04










  • $begingroup$
    @PageDavid: The restriction was meant to be to the subspace $ker(AB)$. Since that subspace was already mentioned a few words later as the domain of the restricted map $tilde B$, I had omitted it to avoid repetition. But you are right, the sentence is more readable when the subspace is explicitly mentioned, so I edited it.
    $endgroup$
    – Marc van Leeuwen
    Mar 29 at 14:25


















10












$begingroup$

As left or right multiplications by invertible matrices (and in particular, elementary matrices) don't change the rank of a matrix, we may assume WLOG that $A=pmatrixI_r&0\ 0&0$. Therefore,
beginalign*
operatornamerank(A)+operatornamerank(B)
&=operatornamerank(A)+operatornamerank(AB+(I-A)B)\
&leoperatornamerank(A)+operatornamerank(AB)+operatornamerank((I-A)B)\
&le r+operatornamerank(AB)+(n-r)\
&=operatornamerank(AB)+n.
endalign*
The last but one line is due to the fact that the first $r$ rows of $(I-A)B$ are zero.






share|cite|improve this answer









$endgroup$












  • $begingroup$
    Nice concise proof !
    $endgroup$
    – Gabriel Romon
    Feb 20 '15 at 8:58










  • $begingroup$
    @Vim No, that's not what I did in the proof (and that equality is false anyway). You see, if $A=PtildeAQ$ where $P,Q$ are invertible and $tildeA=I_roplus0$, the inequality in question is equivalent to $r(tildeA) + r(QB) le r(tildeAQB) + n$. Let $tildeB=QB$, we can rewrite the inequality as $r(tildeA)+r(tildeB)le r(tildeAtildeB)+n$.
    $endgroup$
    – user1551
    Sep 7 '15 at 11:38











  • $begingroup$
    @user1551 I am still a bit confused, what is $I$ in your answer? I don't think it means the identity matrix.
    $endgroup$
    – Vim
    Sep 7 '15 at 12:43










  • $begingroup$
    @Vim It is the identity matrix.
    $endgroup$
    – user1551
    Sep 7 '15 at 14:31










  • $begingroup$
    @user1551 alright.. I thought the sylvester's inequality the OP posted was the general $mtimes stimes s times n$ case.
    $endgroup$
    – Vim
    Sep 7 '15 at 14:42


















9












$begingroup$

$$n+r(AB)=r(M)=r beginpmatrix
I_n & 0 \
0 & AB
endpmatrix$$
Using generalized elementary transformation to $M$:
$$M=beginpmatrix
I_n & 0 \
0 & AB
endpmatrix
to beginpmatrix
I_n & 0 \
A & AB
endpmatrix
to beginpmatrix
I_n & -B \
A & 0
endpmatrix
to beginpmatrix
B & I_n \
0 & A
endpmatrix,$$
hence
$$n+r(AB)=r(M)=
rbeginpmatrix
B & I_n \
0 & A
endpmatrixgeq r(A)+r(B):$$



This solution is from here.






share|cite|improve this answer











$endgroup$




















    4












    $begingroup$

    Denote by $r(A)$ the rank of $A$ and by $n(A)$ the nullity of $A$, i.e. the dimension of the nullspace of $A$. The rank plus nullity theorem says that
    $r(A)+n(A)=n$ and $r(B)+n(B)=n$ and so $2n= r(A)+r(B)+n(A)+n(B)$.



    Let $N(A)$ be the nullspace of $A$ and $R(B)$ the range-space of $B$.
    Then $r(AB)=n - n(B)- dim(N(A) cap R(B))$ (try proving this, hint: what is the nullspace of $AB$?). This gives $n = r(AB) +n(B)+ n(N(A) cap R(B))$ and so $n + r(AB) = r(A)+r(B) +n(A) - n(N(A) cap R(B)) ge
    r(A)+r(B)$.






    share|cite|improve this answer











    $endgroup$




















      2












      $begingroup$

      Let $T:Vto W, S:Wto V$ be linear transformations which correspond to $A,B$. Equivalently we want prove that $r(T)+r(S)leq r(TS)+n$ also similarly show that $$dim ker(T) + dim ker(S) gt dim ker(TS).$$ I know that $ker (S) subset ker (TS)$. Let $a_1,a_2,ldots,a_r$ be a base for $ker (S)$; expand this base for $ker(ST)$ to $a_1,a_2,ldots,a_r,a_r+1,a_r+2,ldots,a_s$ and then show that $S(a_r+1), S(a_r+2),ldots,S(a_s)$ are a set of independent elements s.t. $S(a_i)in ker (T) , forall i:r+1,ldots,s$ and indicate $ker (T) ge s-r,$ so $dim ker T+ dim ker Sge s-r+r=dim ker TS$. $$ 1.dim ker T+ dim ker Sge dim ker TS$$ $$2.dim ker T+r(T)=n$$ $$3.dim ker S T+r(S)=n$$ $$4.dim ker TS+r(TS)=n.$$ By 1, 2, 3, 4 easily conclude $r(T)+r(S)leq r(TS)+n$.






      share|cite|improve this answer











      $endgroup$












      • $begingroup$
        I have reformatted your answer slightly, please check whether it is OK. Also, please allow me to be pedant. You shouldn't denote a base as a set, for instance because the order is important. Mind you, it is perfectly clear what you mean.
        $endgroup$
        – Andreas Caranti
        Feb 9 '13 at 20:02










      • $begingroup$
        @Andreas Caranti: W,V are finite vector space then exist $rin mathbb N$ s.t $a_1,a_2,ldots,a_r$ be base for $ker (S)$
        $endgroup$
        – M.H
        Feb 9 '13 at 20:16










      • $begingroup$
        That's ok, my point was that denoting it as a set $ a_1,a_2,ldots,a_r $ is improper, for instance because if you change the order you get a different base, while in a set there's no order of elements.
        $endgroup$
        – Andreas Caranti
        Feb 9 '13 at 20:54



















      0












      $begingroup$

      In response to Mr. Albanese's correct critique. I submit the following. The rank of A,B is in the set of integers [0,n]. Assume we know the relatively straigtfoward proofs that rank(AB)<= min(rank(A),rank(B)), -> rank(AB) = n if rank(A) and rank(B) have rank n. There are three rank possibilities. Let the rank of A = p, the rank of B = q with p,q integers. Then (p or q < n) or (p and q < n) or (p=q=n). The first two cases imply (rank(A) + rank(B) < rank(AB) + n) or (p+q < min(p,q) + n) as p+q < 2n and (rank(A) + rank(B) > rank(AB) + n) is a contradiction. This is a contrapositive proof of the strict inequality in the Sylvester theorem (rank(A) + rank(B) < rank(AB) + n). For the equality case p=q=n. Then (rank(A) + rank(B) = rank(AB)) + n) or n + n = n + n which is true because rank(A) and rank(B) have full rank, n, implies rank(AB)=n.






      share|cite|improve this answer











      $endgroup$












      • $begingroup$
        This assumes that either $operatornamerank A + operatornamerank B leq operatornamerank AB + n$ or $operatornamerank A + operatornamerank B > operatornamerank AB + n$. It could be the case that there is no relationship between $operatornamerank A + operatornamerank B$ and $operatornamerank AB + n$.
        $endgroup$
        – Michael Albanese
        Sep 7 '14 at 4:32











      • $begingroup$
        I submit the following. The maximum rank of A,B is an integer n.
        $endgroup$
        – John Knox
        Sep 12 '14 at 22:53










      • $begingroup$
        See above per @MichaelAlbanese
        $endgroup$
        – John Knox
        Sep 15 '14 at 22:40











      Your Answer





      StackExchange.ifUsing("editor", function ()
      return StackExchange.using("mathjaxEditing", function ()
      StackExchange.MarkdownEditor.creationCallbacks.add(function (editor, postfix)
      StackExchange.mathjaxEditing.prepareWmdForMathJax(editor, postfix, [["$", "$"], ["\\(","\\)"]]);
      );
      );
      , "mathjax-editing");

      StackExchange.ready(function()
      var channelOptions =
      tags: "".split(" "),
      id: "69"
      ;
      initTagRenderer("".split(" "), "".split(" "), channelOptions);

      StackExchange.using("externalEditor", function()
      // Have to fire editor after snippets, if snippets enabled
      if (StackExchange.settings.snippets.snippetsEnabled)
      StackExchange.using("snippets", function()
      createEditor();
      );

      else
      createEditor();

      );

      function createEditor()
      StackExchange.prepareEditor(
      heartbeatType: 'answer',
      autoActivateHeartbeat: false,
      convertImagesToLinks: true,
      noModals: true,
      showLowRepImageUploadWarning: true,
      reputationToPostImages: 10,
      bindNavPrevention: true,
      postfix: "",
      imageUploader:
      brandingHtml: "Powered by u003ca class="icon-imgur-white" href="https://imgur.com/"u003eu003c/au003e",
      contentPolicyHtml: "User contributions licensed under u003ca href="https://creativecommons.org/licenses/by-sa/3.0/"u003ecc by-sa 3.0 with attribution requiredu003c/au003e u003ca href="https://stackoverflow.com/legal/content-policy"u003e(content policy)u003c/au003e",
      allowUrls: true
      ,
      noCode: true, onDemand: true,
      discardSelector: ".discard-answer"
      ,immediatelyShowMarkdownHelp:true
      );



      );













      draft saved

      draft discarded


















      StackExchange.ready(
      function ()
      StackExchange.openid.initPostLogin('.new-post-login', 'https%3a%2f%2fmath.stackexchange.com%2fquestions%2f298836%2fsylvester-rank-inequality-operatornamerank-a-operatornamerankb-leq-o%23new-answer', 'question_page');

      );

      Post as a guest















      Required, but never shown

























      6 Answers
      6






      active

      oldest

      votes








      6 Answers
      6






      active

      oldest

      votes









      active

      oldest

      votes






      active

      oldest

      votes









      30












      $begingroup$

      Subtract $2n$ from both sides and then multiply by $-1$; this gives the equivalent inequality $defrkoperatornamerank(n-rk A)+(n-rk B)geq n-rk(AB)$. In other words (by the rank-nullity theorem) we must show that the sums of the dimensions of the kernels of $A$ and of $B$ gives at least the dimension of the kernel of $AB$. Intuititively vectors that get killed by $AB$ either get killed by $B$ or (later) by $A$, and the dimension of $ker(AB)$ therefore cannot exceed $dimker A+dimker B$. Here is how to make that precise.



      One has $ker Bsubseteq ker(AB)$, so if one restricts (the linear map with matrix) $B$ to $ker(AB)$, giving a map $tilde B:ker(AB)to K^n$, one sees that $ker(tilde B)=ker(B)$ (both inclusions are obvious). Since the image of $tilde B$ is contained in $ker A$ by the definition of $ker(AB)$, one has $rktilde Bleqdimker A$. Now rank-nullity applied to $tilde B$ gives $$dimker(AB)=dimkertilde B+rktilde Bleqdimker B+dimker A,$$ as desired.






      share|cite|improve this answer











      $endgroup$












      • $begingroup$
        i cant understand how or why you fixed $(n - rank(A)) + (n - rank(B))leq (n - rank(AB)) $ as according to the rank-nullity theorem, this is exactly: $N(A) + N(B) = N(AB)$, i mean, that wouldn't be correct to use $leq$
        $endgroup$
        – Jneven
        Jul 17 '18 at 9:20











      • $begingroup$
        @jneven: As the text says, the new inequality is equivalent to the old one. It still needs to be proved, which I then do.
        $endgroup$
        – Marc van Leeuwen
        Jul 17 '18 at 11:49











      • $begingroup$
        Could you please explain a bit more detail on "restricts matrix to a map"? I searched on the web but only found "restricted to a subspace / domain". Does these have same meaning? Thanks.
        $endgroup$
        – Page David
        Mar 29 at 14:04










      • $begingroup$
        @PageDavid: The restriction was meant to be to the subspace $ker(AB)$. Since that subspace was already mentioned a few words later as the domain of the restricted map $tilde B$, I had omitted it to avoid repetition. But you are right, the sentence is more readable when the subspace is explicitly mentioned, so I edited it.
        $endgroup$
        – Marc van Leeuwen
        Mar 29 at 14:25















      30












      $begingroup$

      Subtract $2n$ from both sides and then multiply by $-1$; this gives the equivalent inequality $defrkoperatornamerank(n-rk A)+(n-rk B)geq n-rk(AB)$. In other words (by the rank-nullity theorem) we must show that the sums of the dimensions of the kernels of $A$ and of $B$ gives at least the dimension of the kernel of $AB$. Intuititively vectors that get killed by $AB$ either get killed by $B$ or (later) by $A$, and the dimension of $ker(AB)$ therefore cannot exceed $dimker A+dimker B$. Here is how to make that precise.



      One has $ker Bsubseteq ker(AB)$, so if one restricts (the linear map with matrix) $B$ to $ker(AB)$, giving a map $tilde B:ker(AB)to K^n$, one sees that $ker(tilde B)=ker(B)$ (both inclusions are obvious). Since the image of $tilde B$ is contained in $ker A$ by the definition of $ker(AB)$, one has $rktilde Bleqdimker A$. Now rank-nullity applied to $tilde B$ gives $$dimker(AB)=dimkertilde B+rktilde Bleqdimker B+dimker A,$$ as desired.






      share|cite|improve this answer











      $endgroup$












      • $begingroup$
        i cant understand how or why you fixed $(n - rank(A)) + (n - rank(B))leq (n - rank(AB)) $ as according to the rank-nullity theorem, this is exactly: $N(A) + N(B) = N(AB)$, i mean, that wouldn't be correct to use $leq$
        $endgroup$
        – Jneven
        Jul 17 '18 at 9:20











      • $begingroup$
        @jneven: As the text says, the new inequality is equivalent to the old one. It still needs to be proved, which I then do.
        $endgroup$
        – Marc van Leeuwen
        Jul 17 '18 at 11:49











      • $begingroup$
        Could you please explain a bit more detail on "restricts matrix to a map"? I searched on the web but only found "restricted to a subspace / domain". Does these have same meaning? Thanks.
        $endgroup$
        – Page David
        Mar 29 at 14:04










      • $begingroup$
        @PageDavid: The restriction was meant to be to the subspace $ker(AB)$. Since that subspace was already mentioned a few words later as the domain of the restricted map $tilde B$, I had omitted it to avoid repetition. But you are right, the sentence is more readable when the subspace is explicitly mentioned, so I edited it.
        $endgroup$
        – Marc van Leeuwen
        Mar 29 at 14:25













      30












      30








      30





      $begingroup$

      Subtract $2n$ from both sides and then multiply by $-1$; this gives the equivalent inequality $defrkoperatornamerank(n-rk A)+(n-rk B)geq n-rk(AB)$. In other words (by the rank-nullity theorem) we must show that the sums of the dimensions of the kernels of $A$ and of $B$ gives at least the dimension of the kernel of $AB$. Intuititively vectors that get killed by $AB$ either get killed by $B$ or (later) by $A$, and the dimension of $ker(AB)$ therefore cannot exceed $dimker A+dimker B$. Here is how to make that precise.



      One has $ker Bsubseteq ker(AB)$, so if one restricts (the linear map with matrix) $B$ to $ker(AB)$, giving a map $tilde B:ker(AB)to K^n$, one sees that $ker(tilde B)=ker(B)$ (both inclusions are obvious). Since the image of $tilde B$ is contained in $ker A$ by the definition of $ker(AB)$, one has $rktilde Bleqdimker A$. Now rank-nullity applied to $tilde B$ gives $$dimker(AB)=dimkertilde B+rktilde Bleqdimker B+dimker A,$$ as desired.






      share|cite|improve this answer











      $endgroup$



      Subtract $2n$ from both sides and then multiply by $-1$; this gives the equivalent inequality $defrkoperatornamerank(n-rk A)+(n-rk B)geq n-rk(AB)$. In other words (by the rank-nullity theorem) we must show that the sums of the dimensions of the kernels of $A$ and of $B$ gives at least the dimension of the kernel of $AB$. Intuititively vectors that get killed by $AB$ either get killed by $B$ or (later) by $A$, and the dimension of $ker(AB)$ therefore cannot exceed $dimker A+dimker B$. Here is how to make that precise.



      One has $ker Bsubseteq ker(AB)$, so if one restricts (the linear map with matrix) $B$ to $ker(AB)$, giving a map $tilde B:ker(AB)to K^n$, one sees that $ker(tilde B)=ker(B)$ (both inclusions are obvious). Since the image of $tilde B$ is contained in $ker A$ by the definition of $ker(AB)$, one has $rktilde Bleqdimker A$. Now rank-nullity applied to $tilde B$ gives $$dimker(AB)=dimkertilde B+rktilde Bleqdimker B+dimker A,$$ as desired.







      share|cite|improve this answer














      share|cite|improve this answer



      share|cite|improve this answer








      edited Mar 29 at 14:23

























      answered Feb 9 '13 at 21:14









      Marc van LeeuwenMarc van Leeuwen

      88.7k5111230




      88.7k5111230











      • $begingroup$
        i cant understand how or why you fixed $(n - rank(A)) + (n - rank(B))leq (n - rank(AB)) $ as according to the rank-nullity theorem, this is exactly: $N(A) + N(B) = N(AB)$, i mean, that wouldn't be correct to use $leq$
        $endgroup$
        – Jneven
        Jul 17 '18 at 9:20











      • $begingroup$
        @jneven: As the text says, the new inequality is equivalent to the old one. It still needs to be proved, which I then do.
        $endgroup$
        – Marc van Leeuwen
        Jul 17 '18 at 11:49











      • $begingroup$
        Could you please explain a bit more detail on "restricts matrix to a map"? I searched on the web but only found "restricted to a subspace / domain". Does these have same meaning? Thanks.
        $endgroup$
        – Page David
        Mar 29 at 14:04










      • $begingroup$
        @PageDavid: The restriction was meant to be to the subspace $ker(AB)$. Since that subspace was already mentioned a few words later as the domain of the restricted map $tilde B$, I had omitted it to avoid repetition. But you are right, the sentence is more readable when the subspace is explicitly mentioned, so I edited it.
        $endgroup$
        – Marc van Leeuwen
        Mar 29 at 14:25
















      • $begingroup$
        i cant understand how or why you fixed $(n - rank(A)) + (n - rank(B))leq (n - rank(AB)) $ as according to the rank-nullity theorem, this is exactly: $N(A) + N(B) = N(AB)$, i mean, that wouldn't be correct to use $leq$
        $endgroup$
        – Jneven
        Jul 17 '18 at 9:20











      • $begingroup$
        @jneven: As the text says, the new inequality is equivalent to the old one. It still needs to be proved, which I then do.
        $endgroup$
        – Marc van Leeuwen
        Jul 17 '18 at 11:49











      • $begingroup$
        Could you please explain a bit more detail on "restricts matrix to a map"? I searched on the web but only found "restricted to a subspace / domain". Does these have same meaning? Thanks.
        $endgroup$
        – Page David
        Mar 29 at 14:04










      • $begingroup$
        @PageDavid: The restriction was meant to be to the subspace $ker(AB)$. Since that subspace was already mentioned a few words later as the domain of the restricted map $tilde B$, I had omitted it to avoid repetition. But you are right, the sentence is more readable when the subspace is explicitly mentioned, so I edited it.
        $endgroup$
        – Marc van Leeuwen
        Mar 29 at 14:25















      $begingroup$
      i cant understand how or why you fixed $(n - rank(A)) + (n - rank(B))leq (n - rank(AB)) $ as according to the rank-nullity theorem, this is exactly: $N(A) + N(B) = N(AB)$, i mean, that wouldn't be correct to use $leq$
      $endgroup$
      – Jneven
      Jul 17 '18 at 9:20





      $begingroup$
      i cant understand how or why you fixed $(n - rank(A)) + (n - rank(B))leq (n - rank(AB)) $ as according to the rank-nullity theorem, this is exactly: $N(A) + N(B) = N(AB)$, i mean, that wouldn't be correct to use $leq$
      $endgroup$
      – Jneven
      Jul 17 '18 at 9:20













      $begingroup$
      @jneven: As the text says, the new inequality is equivalent to the old one. It still needs to be proved, which I then do.
      $endgroup$
      – Marc van Leeuwen
      Jul 17 '18 at 11:49





      $begingroup$
      @jneven: As the text says, the new inequality is equivalent to the old one. It still needs to be proved, which I then do.
      $endgroup$
      – Marc van Leeuwen
      Jul 17 '18 at 11:49













      $begingroup$
      Could you please explain a bit more detail on "restricts matrix to a map"? I searched on the web but only found "restricted to a subspace / domain". Does these have same meaning? Thanks.
      $endgroup$
      – Page David
      Mar 29 at 14:04




      $begingroup$
      Could you please explain a bit more detail on "restricts matrix to a map"? I searched on the web but only found "restricted to a subspace / domain". Does these have same meaning? Thanks.
      $endgroup$
      – Page David
      Mar 29 at 14:04












      $begingroup$
      @PageDavid: The restriction was meant to be to the subspace $ker(AB)$. Since that subspace was already mentioned a few words later as the domain of the restricted map $tilde B$, I had omitted it to avoid repetition. But you are right, the sentence is more readable when the subspace is explicitly mentioned, so I edited it.
      $endgroup$
      – Marc van Leeuwen
      Mar 29 at 14:25




      $begingroup$
      @PageDavid: The restriction was meant to be to the subspace $ker(AB)$. Since that subspace was already mentioned a few words later as the domain of the restricted map $tilde B$, I had omitted it to avoid repetition. But you are right, the sentence is more readable when the subspace is explicitly mentioned, so I edited it.
      $endgroup$
      – Marc van Leeuwen
      Mar 29 at 14:25











      10












      $begingroup$

      As left or right multiplications by invertible matrices (and in particular, elementary matrices) don't change the rank of a matrix, we may assume WLOG that $A=pmatrixI_r&0\ 0&0$. Therefore,
      beginalign*
      operatornamerank(A)+operatornamerank(B)
      &=operatornamerank(A)+operatornamerank(AB+(I-A)B)\
      &leoperatornamerank(A)+operatornamerank(AB)+operatornamerank((I-A)B)\
      &le r+operatornamerank(AB)+(n-r)\
      &=operatornamerank(AB)+n.
      endalign*
      The last but one line is due to the fact that the first $r$ rows of $(I-A)B$ are zero.






      share|cite|improve this answer









      $endgroup$












      • $begingroup$
        Nice concise proof !
        $endgroup$
        – Gabriel Romon
        Feb 20 '15 at 8:58










      • $begingroup$
        @Vim No, that's not what I did in the proof (and that equality is false anyway). You see, if $A=PtildeAQ$ where $P,Q$ are invertible and $tildeA=I_roplus0$, the inequality in question is equivalent to $r(tildeA) + r(QB) le r(tildeAQB) + n$. Let $tildeB=QB$, we can rewrite the inequality as $r(tildeA)+r(tildeB)le r(tildeAtildeB)+n$.
        $endgroup$
        – user1551
        Sep 7 '15 at 11:38











      • $begingroup$
        @user1551 I am still a bit confused, what is $I$ in your answer? I don't think it means the identity matrix.
        $endgroup$
        – Vim
        Sep 7 '15 at 12:43










      • $begingroup$
        @Vim It is the identity matrix.
        $endgroup$
        – user1551
        Sep 7 '15 at 14:31










      • $begingroup$
        @user1551 alright.. I thought the sylvester's inequality the OP posted was the general $mtimes stimes s times n$ case.
        $endgroup$
        – Vim
        Sep 7 '15 at 14:42















      10












      $begingroup$

      As left or right multiplications by invertible matrices (and in particular, elementary matrices) don't change the rank of a matrix, we may assume WLOG that $A=pmatrixI_r&0\ 0&0$. Therefore,
      beginalign*
      operatornamerank(A)+operatornamerank(B)
      &=operatornamerank(A)+operatornamerank(AB+(I-A)B)\
      &leoperatornamerank(A)+operatornamerank(AB)+operatornamerank((I-A)B)\
      &le r+operatornamerank(AB)+(n-r)\
      &=operatornamerank(AB)+n.
      endalign*
      The last but one line is due to the fact that the first $r$ rows of $(I-A)B$ are zero.






      share|cite|improve this answer









      $endgroup$












      • $begingroup$
        Nice concise proof !
        $endgroup$
        – Gabriel Romon
        Feb 20 '15 at 8:58










      • $begingroup$
        @Vim No, that's not what I did in the proof (and that equality is false anyway). You see, if $A=PtildeAQ$ where $P,Q$ are invertible and $tildeA=I_roplus0$, the inequality in question is equivalent to $r(tildeA) + r(QB) le r(tildeAQB) + n$. Let $tildeB=QB$, we can rewrite the inequality as $r(tildeA)+r(tildeB)le r(tildeAtildeB)+n$.
        $endgroup$
        – user1551
        Sep 7 '15 at 11:38











      • $begingroup$
        @user1551 I am still a bit confused, what is $I$ in your answer? I don't think it means the identity matrix.
        $endgroup$
        – Vim
        Sep 7 '15 at 12:43










      • $begingroup$
        @Vim It is the identity matrix.
        $endgroup$
        – user1551
        Sep 7 '15 at 14:31










      • $begingroup$
        @user1551 alright.. I thought the sylvester's inequality the OP posted was the general $mtimes stimes s times n$ case.
        $endgroup$
        – Vim
        Sep 7 '15 at 14:42













      10












      10








      10





      $begingroup$

      As left or right multiplications by invertible matrices (and in particular, elementary matrices) don't change the rank of a matrix, we may assume WLOG that $A=pmatrixI_r&0\ 0&0$. Therefore,
      beginalign*
      operatornamerank(A)+operatornamerank(B)
      &=operatornamerank(A)+operatornamerank(AB+(I-A)B)\
      &leoperatornamerank(A)+operatornamerank(AB)+operatornamerank((I-A)B)\
      &le r+operatornamerank(AB)+(n-r)\
      &=operatornamerank(AB)+n.
      endalign*
      The last but one line is due to the fact that the first $r$ rows of $(I-A)B$ are zero.






      share|cite|improve this answer









      $endgroup$



      As left or right multiplications by invertible matrices (and in particular, elementary matrices) don't change the rank of a matrix, we may assume WLOG that $A=pmatrixI_r&0\ 0&0$. Therefore,
      beginalign*
      operatornamerank(A)+operatornamerank(B)
      &=operatornamerank(A)+operatornamerank(AB+(I-A)B)\
      &leoperatornamerank(A)+operatornamerank(AB)+operatornamerank((I-A)B)\
      &le r+operatornamerank(AB)+(n-r)\
      &=operatornamerank(AB)+n.
      endalign*
      The last but one line is due to the fact that the first $r$ rows of $(I-A)B$ are zero.







      share|cite|improve this answer












      share|cite|improve this answer



      share|cite|improve this answer










      answered Apr 4 '13 at 6:11









      user1551user1551

      74k566129




      74k566129











      • $begingroup$
        Nice concise proof !
        $endgroup$
        – Gabriel Romon
        Feb 20 '15 at 8:58










      • $begingroup$
        @Vim No, that's not what I did in the proof (and that equality is false anyway). You see, if $A=PtildeAQ$ where $P,Q$ are invertible and $tildeA=I_roplus0$, the inequality in question is equivalent to $r(tildeA) + r(QB) le r(tildeAQB) + n$. Let $tildeB=QB$, we can rewrite the inequality as $r(tildeA)+r(tildeB)le r(tildeAtildeB)+n$.
        $endgroup$
        – user1551
        Sep 7 '15 at 11:38











      • $begingroup$
        @user1551 I am still a bit confused, what is $I$ in your answer? I don't think it means the identity matrix.
        $endgroup$
        – Vim
        Sep 7 '15 at 12:43










      • $begingroup$
        @Vim It is the identity matrix.
        $endgroup$
        – user1551
        Sep 7 '15 at 14:31










      • $begingroup$
        @user1551 alright.. I thought the sylvester's inequality the OP posted was the general $mtimes stimes s times n$ case.
        $endgroup$
        – Vim
        Sep 7 '15 at 14:42
















      • $begingroup$
        Nice concise proof !
        $endgroup$
        – Gabriel Romon
        Feb 20 '15 at 8:58










      • $begingroup$
        @Vim No, that's not what I did in the proof (and that equality is false anyway). You see, if $A=PtildeAQ$ where $P,Q$ are invertible and $tildeA=I_roplus0$, the inequality in question is equivalent to $r(tildeA) + r(QB) le r(tildeAQB) + n$. Let $tildeB=QB$, we can rewrite the inequality as $r(tildeA)+r(tildeB)le r(tildeAtildeB)+n$.
        $endgroup$
        – user1551
        Sep 7 '15 at 11:38











      • $begingroup$
        @user1551 I am still a bit confused, what is $I$ in your answer? I don't think it means the identity matrix.
        $endgroup$
        – Vim
        Sep 7 '15 at 12:43










      • $begingroup$
        @Vim It is the identity matrix.
        $endgroup$
        – user1551
        Sep 7 '15 at 14:31










      • $begingroup$
        @user1551 alright.. I thought the sylvester's inequality the OP posted was the general $mtimes stimes s times n$ case.
        $endgroup$
        – Vim
        Sep 7 '15 at 14:42















      $begingroup$
      Nice concise proof !
      $endgroup$
      – Gabriel Romon
      Feb 20 '15 at 8:58




      $begingroup$
      Nice concise proof !
      $endgroup$
      – Gabriel Romon
      Feb 20 '15 at 8:58












      $begingroup$
      @Vim No, that's not what I did in the proof (and that equality is false anyway). You see, if $A=PtildeAQ$ where $P,Q$ are invertible and $tildeA=I_roplus0$, the inequality in question is equivalent to $r(tildeA) + r(QB) le r(tildeAQB) + n$. Let $tildeB=QB$, we can rewrite the inequality as $r(tildeA)+r(tildeB)le r(tildeAtildeB)+n$.
      $endgroup$
      – user1551
      Sep 7 '15 at 11:38





      $begingroup$
      @Vim No, that's not what I did in the proof (and that equality is false anyway). You see, if $A=PtildeAQ$ where $P,Q$ are invertible and $tildeA=I_roplus0$, the inequality in question is equivalent to $r(tildeA) + r(QB) le r(tildeAQB) + n$. Let $tildeB=QB$, we can rewrite the inequality as $r(tildeA)+r(tildeB)le r(tildeAtildeB)+n$.
      $endgroup$
      – user1551
      Sep 7 '15 at 11:38













      $begingroup$
      @user1551 I am still a bit confused, what is $I$ in your answer? I don't think it means the identity matrix.
      $endgroup$
      – Vim
      Sep 7 '15 at 12:43




      $begingroup$
      @user1551 I am still a bit confused, what is $I$ in your answer? I don't think it means the identity matrix.
      $endgroup$
      – Vim
      Sep 7 '15 at 12:43












      $begingroup$
      @Vim It is the identity matrix.
      $endgroup$
      – user1551
      Sep 7 '15 at 14:31




      $begingroup$
      @Vim It is the identity matrix.
      $endgroup$
      – user1551
      Sep 7 '15 at 14:31












      $begingroup$
      @user1551 alright.. I thought the sylvester's inequality the OP posted was the general $mtimes stimes s times n$ case.
      $endgroup$
      – Vim
      Sep 7 '15 at 14:42




      $begingroup$
      @user1551 alright.. I thought the sylvester's inequality the OP posted was the general $mtimes stimes s times n$ case.
      $endgroup$
      – Vim
      Sep 7 '15 at 14:42











      9












      $begingroup$

      $$n+r(AB)=r(M)=r beginpmatrix
      I_n & 0 \
      0 & AB
      endpmatrix$$
      Using generalized elementary transformation to $M$:
      $$M=beginpmatrix
      I_n & 0 \
      0 & AB
      endpmatrix
      to beginpmatrix
      I_n & 0 \
      A & AB
      endpmatrix
      to beginpmatrix
      I_n & -B \
      A & 0
      endpmatrix
      to beginpmatrix
      B & I_n \
      0 & A
      endpmatrix,$$
      hence
      $$n+r(AB)=r(M)=
      rbeginpmatrix
      B & I_n \
      0 & A
      endpmatrixgeq r(A)+r(B):$$



      This solution is from here.






      share|cite|improve this answer











      $endgroup$

















        9












        $begingroup$

        $$n+r(AB)=r(M)=r beginpmatrix
        I_n & 0 \
        0 & AB
        endpmatrix$$
        Using generalized elementary transformation to $M$:
        $$M=beginpmatrix
        I_n & 0 \
        0 & AB
        endpmatrix
        to beginpmatrix
        I_n & 0 \
        A & AB
        endpmatrix
        to beginpmatrix
        I_n & -B \
        A & 0
        endpmatrix
        to beginpmatrix
        B & I_n \
        0 & A
        endpmatrix,$$
        hence
        $$n+r(AB)=r(M)=
        rbeginpmatrix
        B & I_n \
        0 & A
        endpmatrixgeq r(A)+r(B):$$



        This solution is from here.






        share|cite|improve this answer











        $endgroup$















          9












          9








          9





          $begingroup$

          $$n+r(AB)=r(M)=r beginpmatrix
          I_n & 0 \
          0 & AB
          endpmatrix$$
          Using generalized elementary transformation to $M$:
          $$M=beginpmatrix
          I_n & 0 \
          0 & AB
          endpmatrix
          to beginpmatrix
          I_n & 0 \
          A & AB
          endpmatrix
          to beginpmatrix
          I_n & -B \
          A & 0
          endpmatrix
          to beginpmatrix
          B & I_n \
          0 & A
          endpmatrix,$$
          hence
          $$n+r(AB)=r(M)=
          rbeginpmatrix
          B & I_n \
          0 & A
          endpmatrixgeq r(A)+r(B):$$



          This solution is from here.






          share|cite|improve this answer











          $endgroup$



          $$n+r(AB)=r(M)=r beginpmatrix
          I_n & 0 \
          0 & AB
          endpmatrix$$
          Using generalized elementary transformation to $M$:
          $$M=beginpmatrix
          I_n & 0 \
          0 & AB
          endpmatrix
          to beginpmatrix
          I_n & 0 \
          A & AB
          endpmatrix
          to beginpmatrix
          I_n & -B \
          A & 0
          endpmatrix
          to beginpmatrix
          B & I_n \
          0 & A
          endpmatrix,$$
          hence
          $$n+r(AB)=r(M)=
          rbeginpmatrix
          B & I_n \
          0 & A
          endpmatrixgeq r(A)+r(B):$$



          This solution is from here.







          share|cite|improve this answer














          share|cite|improve this answer



          share|cite|improve this answer








          edited Feb 20 '15 at 8:50

























          answered Jul 8 '13 at 10:24









          MherMher

          3,8301432




          3,8301432





















              4












              $begingroup$

              Denote by $r(A)$ the rank of $A$ and by $n(A)$ the nullity of $A$, i.e. the dimension of the nullspace of $A$. The rank plus nullity theorem says that
              $r(A)+n(A)=n$ and $r(B)+n(B)=n$ and so $2n= r(A)+r(B)+n(A)+n(B)$.



              Let $N(A)$ be the nullspace of $A$ and $R(B)$ the range-space of $B$.
              Then $r(AB)=n - n(B)- dim(N(A) cap R(B))$ (try proving this, hint: what is the nullspace of $AB$?). This gives $n = r(AB) +n(B)+ n(N(A) cap R(B))$ and so $n + r(AB) = r(A)+r(B) +n(A) - n(N(A) cap R(B)) ge
              r(A)+r(B)$.






              share|cite|improve this answer











              $endgroup$

















                4












                $begingroup$

                Denote by $r(A)$ the rank of $A$ and by $n(A)$ the nullity of $A$, i.e. the dimension of the nullspace of $A$. The rank plus nullity theorem says that
                $r(A)+n(A)=n$ and $r(B)+n(B)=n$ and so $2n= r(A)+r(B)+n(A)+n(B)$.



                Let $N(A)$ be the nullspace of $A$ and $R(B)$ the range-space of $B$.
                Then $r(AB)=n - n(B)- dim(N(A) cap R(B))$ (try proving this, hint: what is the nullspace of $AB$?). This gives $n = r(AB) +n(B)+ n(N(A) cap R(B))$ and so $n + r(AB) = r(A)+r(B) +n(A) - n(N(A) cap R(B)) ge
                r(A)+r(B)$.






                share|cite|improve this answer











                $endgroup$















                  4












                  4








                  4





                  $begingroup$

                  Denote by $r(A)$ the rank of $A$ and by $n(A)$ the nullity of $A$, i.e. the dimension of the nullspace of $A$. The rank plus nullity theorem says that
                  $r(A)+n(A)=n$ and $r(B)+n(B)=n$ and so $2n= r(A)+r(B)+n(A)+n(B)$.



                  Let $N(A)$ be the nullspace of $A$ and $R(B)$ the range-space of $B$.
                  Then $r(AB)=n - n(B)- dim(N(A) cap R(B))$ (try proving this, hint: what is the nullspace of $AB$?). This gives $n = r(AB) +n(B)+ n(N(A) cap R(B))$ and so $n + r(AB) = r(A)+r(B) +n(A) - n(N(A) cap R(B)) ge
                  r(A)+r(B)$.






                  share|cite|improve this answer











                  $endgroup$



                  Denote by $r(A)$ the rank of $A$ and by $n(A)$ the nullity of $A$, i.e. the dimension of the nullspace of $A$. The rank plus nullity theorem says that
                  $r(A)+n(A)=n$ and $r(B)+n(B)=n$ and so $2n= r(A)+r(B)+n(A)+n(B)$.



                  Let $N(A)$ be the nullspace of $A$ and $R(B)$ the range-space of $B$.
                  Then $r(AB)=n - n(B)- dim(N(A) cap R(B))$ (try proving this, hint: what is the nullspace of $AB$?). This gives $n = r(AB) +n(B)+ n(N(A) cap R(B))$ and so $n + r(AB) = r(A)+r(B) +n(A) - n(N(A) cap R(B)) ge
                  r(A)+r(B)$.







                  share|cite|improve this answer














                  share|cite|improve this answer



                  share|cite|improve this answer








                  edited Apr 2 '13 at 15:46







                  user26857

















                  answered Feb 9 '13 at 20:08









                  ManosManos

                  14.1k33288




                  14.1k33288





















                      2












                      $begingroup$

                      Let $T:Vto W, S:Wto V$ be linear transformations which correspond to $A,B$. Equivalently we want prove that $r(T)+r(S)leq r(TS)+n$ also similarly show that $$dim ker(T) + dim ker(S) gt dim ker(TS).$$ I know that $ker (S) subset ker (TS)$. Let $a_1,a_2,ldots,a_r$ be a base for $ker (S)$; expand this base for $ker(ST)$ to $a_1,a_2,ldots,a_r,a_r+1,a_r+2,ldots,a_s$ and then show that $S(a_r+1), S(a_r+2),ldots,S(a_s)$ are a set of independent elements s.t. $S(a_i)in ker (T) , forall i:r+1,ldots,s$ and indicate $ker (T) ge s-r,$ so $dim ker T+ dim ker Sge s-r+r=dim ker TS$. $$ 1.dim ker T+ dim ker Sge dim ker TS$$ $$2.dim ker T+r(T)=n$$ $$3.dim ker S T+r(S)=n$$ $$4.dim ker TS+r(TS)=n.$$ By 1, 2, 3, 4 easily conclude $r(T)+r(S)leq r(TS)+n$.






                      share|cite|improve this answer











                      $endgroup$












                      • $begingroup$
                        I have reformatted your answer slightly, please check whether it is OK. Also, please allow me to be pedant. You shouldn't denote a base as a set, for instance because the order is important. Mind you, it is perfectly clear what you mean.
                        $endgroup$
                        – Andreas Caranti
                        Feb 9 '13 at 20:02










                      • $begingroup$
                        @Andreas Caranti: W,V are finite vector space then exist $rin mathbb N$ s.t $a_1,a_2,ldots,a_r$ be base for $ker (S)$
                        $endgroup$
                        – M.H
                        Feb 9 '13 at 20:16










                      • $begingroup$
                        That's ok, my point was that denoting it as a set $ a_1,a_2,ldots,a_r $ is improper, for instance because if you change the order you get a different base, while in a set there's no order of elements.
                        $endgroup$
                        – Andreas Caranti
                        Feb 9 '13 at 20:54
















                      2












                      $begingroup$

                      Let $T:Vto W, S:Wto V$ be linear transformations which correspond to $A,B$. Equivalently we want prove that $r(T)+r(S)leq r(TS)+n$ also similarly show that $$dim ker(T) + dim ker(S) gt dim ker(TS).$$ I know that $ker (S) subset ker (TS)$. Let $a_1,a_2,ldots,a_r$ be a base for $ker (S)$; expand this base for $ker(ST)$ to $a_1,a_2,ldots,a_r,a_r+1,a_r+2,ldots,a_s$ and then show that $S(a_r+1), S(a_r+2),ldots,S(a_s)$ are a set of independent elements s.t. $S(a_i)in ker (T) , forall i:r+1,ldots,s$ and indicate $ker (T) ge s-r,$ so $dim ker T+ dim ker Sge s-r+r=dim ker TS$. $$ 1.dim ker T+ dim ker Sge dim ker TS$$ $$2.dim ker T+r(T)=n$$ $$3.dim ker S T+r(S)=n$$ $$4.dim ker TS+r(TS)=n.$$ By 1, 2, 3, 4 easily conclude $r(T)+r(S)leq r(TS)+n$.






                      share|cite|improve this answer











                      $endgroup$












                      • $begingroup$
                        I have reformatted your answer slightly, please check whether it is OK. Also, please allow me to be pedant. You shouldn't denote a base as a set, for instance because the order is important. Mind you, it is perfectly clear what you mean.
                        $endgroup$
                        – Andreas Caranti
                        Feb 9 '13 at 20:02










                      • $begingroup$
                        @Andreas Caranti: W,V are finite vector space then exist $rin mathbb N$ s.t $a_1,a_2,ldots,a_r$ be base for $ker (S)$
                        $endgroup$
                        – M.H
                        Feb 9 '13 at 20:16










                      • $begingroup$
                        That's ok, my point was that denoting it as a set $ a_1,a_2,ldots,a_r $ is improper, for instance because if you change the order you get a different base, while in a set there's no order of elements.
                        $endgroup$
                        – Andreas Caranti
                        Feb 9 '13 at 20:54














                      2












                      2








                      2





                      $begingroup$

                      Let $T:Vto W, S:Wto V$ be linear transformations which correspond to $A,B$. Equivalently we want prove that $r(T)+r(S)leq r(TS)+n$ also similarly show that $$dim ker(T) + dim ker(S) gt dim ker(TS).$$ I know that $ker (S) subset ker (TS)$. Let $a_1,a_2,ldots,a_r$ be a base for $ker (S)$; expand this base for $ker(ST)$ to $a_1,a_2,ldots,a_r,a_r+1,a_r+2,ldots,a_s$ and then show that $S(a_r+1), S(a_r+2),ldots,S(a_s)$ are a set of independent elements s.t. $S(a_i)in ker (T) , forall i:r+1,ldots,s$ and indicate $ker (T) ge s-r,$ so $dim ker T+ dim ker Sge s-r+r=dim ker TS$. $$ 1.dim ker T+ dim ker Sge dim ker TS$$ $$2.dim ker T+r(T)=n$$ $$3.dim ker S T+r(S)=n$$ $$4.dim ker TS+r(TS)=n.$$ By 1, 2, 3, 4 easily conclude $r(T)+r(S)leq r(TS)+n$.






                      share|cite|improve this answer











                      $endgroup$



                      Let $T:Vto W, S:Wto V$ be linear transformations which correspond to $A,B$. Equivalently we want prove that $r(T)+r(S)leq r(TS)+n$ also similarly show that $$dim ker(T) + dim ker(S) gt dim ker(TS).$$ I know that $ker (S) subset ker (TS)$. Let $a_1,a_2,ldots,a_r$ be a base for $ker (S)$; expand this base for $ker(ST)$ to $a_1,a_2,ldots,a_r,a_r+1,a_r+2,ldots,a_s$ and then show that $S(a_r+1), S(a_r+2),ldots,S(a_s)$ are a set of independent elements s.t. $S(a_i)in ker (T) , forall i:r+1,ldots,s$ and indicate $ker (T) ge s-r,$ so $dim ker T+ dim ker Sge s-r+r=dim ker TS$. $$ 1.dim ker T+ dim ker Sge dim ker TS$$ $$2.dim ker T+r(T)=n$$ $$3.dim ker S T+r(S)=n$$ $$4.dim ker TS+r(TS)=n.$$ By 1, 2, 3, 4 easily conclude $r(T)+r(S)leq r(TS)+n$.







                      share|cite|improve this answer














                      share|cite|improve this answer



                      share|cite|improve this answer








                      edited Apr 2 '13 at 15:55







                      user26857

















                      answered Feb 9 '13 at 19:56









                      M.HM.H

                      7,32211654




                      7,32211654











                      • $begingroup$
                        I have reformatted your answer slightly, please check whether it is OK. Also, please allow me to be pedant. You shouldn't denote a base as a set, for instance because the order is important. Mind you, it is perfectly clear what you mean.
                        $endgroup$
                        – Andreas Caranti
                        Feb 9 '13 at 20:02










                      • $begingroup$
                        @Andreas Caranti: W,V are finite vector space then exist $rin mathbb N$ s.t $a_1,a_2,ldots,a_r$ be base for $ker (S)$
                        $endgroup$
                        – M.H
                        Feb 9 '13 at 20:16










                      • $begingroup$
                        That's ok, my point was that denoting it as a set $ a_1,a_2,ldots,a_r $ is improper, for instance because if you change the order you get a different base, while in a set there's no order of elements.
                        $endgroup$
                        – Andreas Caranti
                        Feb 9 '13 at 20:54

















                      • $begingroup$
                        I have reformatted your answer slightly, please check whether it is OK. Also, please allow me to be pedant. You shouldn't denote a base as a set, for instance because the order is important. Mind you, it is perfectly clear what you mean.
                        $endgroup$
                        – Andreas Caranti
                        Feb 9 '13 at 20:02










                      • $begingroup$
                        @Andreas Caranti: W,V are finite vector space then exist $rin mathbb N$ s.t $a_1,a_2,ldots,a_r$ be base for $ker (S)$
                        $endgroup$
                        – M.H
                        Feb 9 '13 at 20:16










                      • $begingroup$
                        That's ok, my point was that denoting it as a set $ a_1,a_2,ldots,a_r $ is improper, for instance because if you change the order you get a different base, while in a set there's no order of elements.
                        $endgroup$
                        – Andreas Caranti
                        Feb 9 '13 at 20:54
















                      $begingroup$
                      I have reformatted your answer slightly, please check whether it is OK. Also, please allow me to be pedant. You shouldn't denote a base as a set, for instance because the order is important. Mind you, it is perfectly clear what you mean.
                      $endgroup$
                      – Andreas Caranti
                      Feb 9 '13 at 20:02




                      $begingroup$
                      I have reformatted your answer slightly, please check whether it is OK. Also, please allow me to be pedant. You shouldn't denote a base as a set, for instance because the order is important. Mind you, it is perfectly clear what you mean.
                      $endgroup$
                      – Andreas Caranti
                      Feb 9 '13 at 20:02












                      $begingroup$
                      @Andreas Caranti: W,V are finite vector space then exist $rin mathbb N$ s.t $a_1,a_2,ldots,a_r$ be base for $ker (S)$
                      $endgroup$
                      – M.H
                      Feb 9 '13 at 20:16




                      $begingroup$
                      @Andreas Caranti: W,V are finite vector space then exist $rin mathbb N$ s.t $a_1,a_2,ldots,a_r$ be base for $ker (S)$
                      $endgroup$
                      – M.H
                      Feb 9 '13 at 20:16












                      $begingroup$
                      That's ok, my point was that denoting it as a set $ a_1,a_2,ldots,a_r $ is improper, for instance because if you change the order you get a different base, while in a set there's no order of elements.
                      $endgroup$
                      – Andreas Caranti
                      Feb 9 '13 at 20:54





                      $begingroup$
                      That's ok, my point was that denoting it as a set $ a_1,a_2,ldots,a_r $ is improper, for instance because if you change the order you get a different base, while in a set there's no order of elements.
                      $endgroup$
                      – Andreas Caranti
                      Feb 9 '13 at 20:54












                      0












                      $begingroup$

                      In response to Mr. Albanese's correct critique. I submit the following. The rank of A,B is in the set of integers [0,n]. Assume we know the relatively straigtfoward proofs that rank(AB)<= min(rank(A),rank(B)), -> rank(AB) = n if rank(A) and rank(B) have rank n. There are three rank possibilities. Let the rank of A = p, the rank of B = q with p,q integers. Then (p or q < n) or (p and q < n) or (p=q=n). The first two cases imply (rank(A) + rank(B) < rank(AB) + n) or (p+q < min(p,q) + n) as p+q < 2n and (rank(A) + rank(B) > rank(AB) + n) is a contradiction. This is a contrapositive proof of the strict inequality in the Sylvester theorem (rank(A) + rank(B) < rank(AB) + n). For the equality case p=q=n. Then (rank(A) + rank(B) = rank(AB)) + n) or n + n = n + n which is true because rank(A) and rank(B) have full rank, n, implies rank(AB)=n.






                      share|cite|improve this answer











                      $endgroup$












                      • $begingroup$
                        This assumes that either $operatornamerank A + operatornamerank B leq operatornamerank AB + n$ or $operatornamerank A + operatornamerank B > operatornamerank AB + n$. It could be the case that there is no relationship between $operatornamerank A + operatornamerank B$ and $operatornamerank AB + n$.
                        $endgroup$
                        – Michael Albanese
                        Sep 7 '14 at 4:32











                      • $begingroup$
                        I submit the following. The maximum rank of A,B is an integer n.
                        $endgroup$
                        – John Knox
                        Sep 12 '14 at 22:53










                      • $begingroup$
                        See above per @MichaelAlbanese
                        $endgroup$
                        – John Knox
                        Sep 15 '14 at 22:40















                      0












                      $begingroup$

                      In response to Mr. Albanese's correct critique. I submit the following. The rank of A,B is in the set of integers [0,n]. Assume we know the relatively straigtfoward proofs that rank(AB)<= min(rank(A),rank(B)), -> rank(AB) = n if rank(A) and rank(B) have rank n. There are three rank possibilities. Let the rank of A = p, the rank of B = q with p,q integers. Then (p or q < n) or (p and q < n) or (p=q=n). The first two cases imply (rank(A) + rank(B) < rank(AB) + n) or (p+q < min(p,q) + n) as p+q < 2n and (rank(A) + rank(B) > rank(AB) + n) is a contradiction. This is a contrapositive proof of the strict inequality in the Sylvester theorem (rank(A) + rank(B) < rank(AB) + n). For the equality case p=q=n. Then (rank(A) + rank(B) = rank(AB)) + n) or n + n = n + n which is true because rank(A) and rank(B) have full rank, n, implies rank(AB)=n.






                      share|cite|improve this answer











                      $endgroup$












                      • $begingroup$
                        This assumes that either $operatornamerank A + operatornamerank B leq operatornamerank AB + n$ or $operatornamerank A + operatornamerank B > operatornamerank AB + n$. It could be the case that there is no relationship between $operatornamerank A + operatornamerank B$ and $operatornamerank AB + n$.
                        $endgroup$
                        – Michael Albanese
                        Sep 7 '14 at 4:32











                      • $begingroup$
                        I submit the following. The maximum rank of A,B is an integer n.
                        $endgroup$
                        – John Knox
                        Sep 12 '14 at 22:53










                      • $begingroup$
                        See above per @MichaelAlbanese
                        $endgroup$
                        – John Knox
                        Sep 15 '14 at 22:40













                      0












                      0








                      0





                      $begingroup$

                      In response to Mr. Albanese's correct critique. I submit the following. The rank of A,B is in the set of integers [0,n]. Assume we know the relatively straigtfoward proofs that rank(AB)<= min(rank(A),rank(B)), -> rank(AB) = n if rank(A) and rank(B) have rank n. There are three rank possibilities. Let the rank of A = p, the rank of B = q with p,q integers. Then (p or q < n) or (p and q < n) or (p=q=n). The first two cases imply (rank(A) + rank(B) < rank(AB) + n) or (p+q < min(p,q) + n) as p+q < 2n and (rank(A) + rank(B) > rank(AB) + n) is a contradiction. This is a contrapositive proof of the strict inequality in the Sylvester theorem (rank(A) + rank(B) < rank(AB) + n). For the equality case p=q=n. Then (rank(A) + rank(B) = rank(AB)) + n) or n + n = n + n which is true because rank(A) and rank(B) have full rank, n, implies rank(AB)=n.






                      share|cite|improve this answer











                      $endgroup$



                      In response to Mr. Albanese's correct critique. I submit the following. The rank of A,B is in the set of integers [0,n]. Assume we know the relatively straigtfoward proofs that rank(AB)<= min(rank(A),rank(B)), -> rank(AB) = n if rank(A) and rank(B) have rank n. There are three rank possibilities. Let the rank of A = p, the rank of B = q with p,q integers. Then (p or q < n) or (p and q < n) or (p=q=n). The first two cases imply (rank(A) + rank(B) < rank(AB) + n) or (p+q < min(p,q) + n) as p+q < 2n and (rank(A) + rank(B) > rank(AB) + n) is a contradiction. This is a contrapositive proof of the strict inequality in the Sylvester theorem (rank(A) + rank(B) < rank(AB) + n). For the equality case p=q=n. Then (rank(A) + rank(B) = rank(AB)) + n) or n + n = n + n which is true because rank(A) and rank(B) have full rank, n, implies rank(AB)=n.







                      share|cite|improve this answer














                      share|cite|improve this answer



                      share|cite|improve this answer








                      edited Sep 13 '14 at 3:07

























                      answered Sep 7 '14 at 4:10









                      John KnoxJohn Knox

                      114




                      114











                      • $begingroup$
                        This assumes that either $operatornamerank A + operatornamerank B leq operatornamerank AB + n$ or $operatornamerank A + operatornamerank B > operatornamerank AB + n$. It could be the case that there is no relationship between $operatornamerank A + operatornamerank B$ and $operatornamerank AB + n$.
                        $endgroup$
                        – Michael Albanese
                        Sep 7 '14 at 4:32











                      • $begingroup$
                        I submit the following. The maximum rank of A,B is an integer n.
                        $endgroup$
                        – John Knox
                        Sep 12 '14 at 22:53










                      • $begingroup$
                        See above per @MichaelAlbanese
                        $endgroup$
                        – John Knox
                        Sep 15 '14 at 22:40
















                      • $begingroup$
                        This assumes that either $operatornamerank A + operatornamerank B leq operatornamerank AB + n$ or $operatornamerank A + operatornamerank B > operatornamerank AB + n$. It could be the case that there is no relationship between $operatornamerank A + operatornamerank B$ and $operatornamerank AB + n$.
                        $endgroup$
                        – Michael Albanese
                        Sep 7 '14 at 4:32











                      • $begingroup$
                        I submit the following. The maximum rank of A,B is an integer n.
                        $endgroup$
                        – John Knox
                        Sep 12 '14 at 22:53










                      • $begingroup$
                        See above per @MichaelAlbanese
                        $endgroup$
                        – John Knox
                        Sep 15 '14 at 22:40















                      $begingroup$
                      This assumes that either $operatornamerank A + operatornamerank B leq operatornamerank AB + n$ or $operatornamerank A + operatornamerank B > operatornamerank AB + n$. It could be the case that there is no relationship between $operatornamerank A + operatornamerank B$ and $operatornamerank AB + n$.
                      $endgroup$
                      – Michael Albanese
                      Sep 7 '14 at 4:32





                      $begingroup$
                      This assumes that either $operatornamerank A + operatornamerank B leq operatornamerank AB + n$ or $operatornamerank A + operatornamerank B > operatornamerank AB + n$. It could be the case that there is no relationship between $operatornamerank A + operatornamerank B$ and $operatornamerank AB + n$.
                      $endgroup$
                      – Michael Albanese
                      Sep 7 '14 at 4:32













                      $begingroup$
                      I submit the following. The maximum rank of A,B is an integer n.
                      $endgroup$
                      – John Knox
                      Sep 12 '14 at 22:53




                      $begingroup$
                      I submit the following. The maximum rank of A,B is an integer n.
                      $endgroup$
                      – John Knox
                      Sep 12 '14 at 22:53












                      $begingroup$
                      See above per @MichaelAlbanese
                      $endgroup$
                      – John Knox
                      Sep 15 '14 at 22:40




                      $begingroup$
                      See above per @MichaelAlbanese
                      $endgroup$
                      – John Knox
                      Sep 15 '14 at 22:40

















                      draft saved

                      draft discarded
















































                      Thanks for contributing an answer to Mathematics Stack Exchange!


                      • Please be sure to answer the question. Provide details and share your research!

                      But avoid


                      • Asking for help, clarification, or responding to other answers.

                      • Making statements based on opinion; back them up with references or personal experience.

                      Use MathJax to format equations. MathJax reference.


                      To learn more, see our tips on writing great answers.




                      draft saved


                      draft discarded














                      StackExchange.ready(
                      function ()
                      StackExchange.openid.initPostLogin('.new-post-login', 'https%3a%2f%2fmath.stackexchange.com%2fquestions%2f298836%2fsylvester-rank-inequality-operatornamerank-a-operatornamerankb-leq-o%23new-answer', 'question_page');

                      );

                      Post as a guest















                      Required, but never shown





















































                      Required, but never shown














                      Required, but never shown












                      Required, but never shown







                      Required, but never shown

































                      Required, but never shown














                      Required, but never shown












                      Required, but never shown







                      Required, but never shown







                      Popular posts from this blog

                      Triangular numbers and gcdProving sum of a set is $0 pmod n$ if $n$ is odd, or $fracn2 pmod n$ if $n$ is even?Is greatest common divisor of two numbers really their smallest linear combination?GCD, LCM RelationshipProve a set of nonnegative integers with greatest common divisor 1 and closed under addition has all but finite many nonnegative integers.all pairs of a and b in an equation containing gcdTriangular Numbers Modulo $k$ - Hit All Values?Understanding the Existence and Uniqueness of the GCDGCD and LCM with logical symbolsThe greatest common divisor of two positive integers less than 100 is equal to 3. Their least common multiple is twelve times one of the integers.Suppose that for all integers $x$, $x|a$ and $x|b$ if and only if $x|c$. Then $c = gcd(a,b)$Which is the gcd of 2 numbers which are multiplied and the result is 600000?

                      Ingelân Ynhâld Etymology | Geografy | Skiednis | Polityk en bestjoer | Ekonomy | Demografy | Kultuer | Klimaat | Sjoch ek | Keppelings om utens | Boarnen, noaten en referinsjes Navigaasjemenuwww.gov.ukOffisjele webside fan it regear fan it Feriene KeninkrykOffisjele webside fan it Britske FerkearsburoNederlânsktalige ynformaasje fan it Britske FerkearsburoOffisjele webside fan English Heritage, de organisaasje dy't him ynset foar it behâld fan it Ingelske kultuergoedYnwennertallen fan alle Britske stêden út 'e folkstelling fan 2011Notes en References, op dizze sideEngland

                      Հադիս Բովանդակություն Անվանում և նշանակություն | Դասակարգում | Աղբյուրներ | Նավարկման ցանկ